What is the slope of the points (3,64) and (9,79).
m=
m =
= 15
6
m =
Un Hồ
2-#1
m=2.5
6
15

What Is The Slope Of The Points (3,64) And (9,79).m=m == 156m =Un H2-#1m=2.5615

Answers

Answer 1

Answer:

[tex]\boxed{\bf Slope(m)=2.5}[/tex]

Step-by-step explanation:

We can use the slope formula to find the slope of a line given the coordinates of two points on the line:- (3,64) and (9,79).

The coordinates of the first point represent x_1 and y_1. The coordinates of the second points are x_2, y_2.

[tex]\boxed{\bf \mathrm{Slope}=\cfrac{y_2-y_1}{x_2-x_1}}[/tex]

[tex]\sf \left(x_1,\:y_1\right)=\left(3,\:64\right)[/tex]

[tex]\sf \:\left(x_2,\:y_2\right)=\left(9,\:79\right)[/tex]

[tex]\sf m=\cfrac{79-64}{9-3}[/tex]

[tex]\sf m=\cfrac{5}{2}[/tex]

[tex]\sf m=2.5[/tex]

Therefore, the slope of (3,64) and (9,79) is D) 2.5!!

___________________

Hope this helps!
Have a great day!

Answer 2

Answer:

m = (y2 - y1)/(x2 - x1) m = 15/6 m = 2.5

Step-by-step explanation:

Formula we use,

→ m = (y2 - y1)/(x2 - x1)

Then the required slope is,

→ m = (y2 - y1)/(x2 - x1)

→ m = (79 - 64)/(9 - 3)

→ m = 15/6

→ [ m = 2.5 ]

Hence, the slope is 2.5.


Related Questions

Thomas is married and files jointly with his spouse. Their combined taxable income is $25,799. Their employers withheld $4,386 in taxesfor the year. Determine theamount to be refundedor the balance due.Circle one: RefundBalance Due

Answers

EXPLANATION

As we can see on the table, the amount to be refunded is equivalent to the difference between $3,866 and $4,386, so it is $520

Write an expression for the operation described.

"5 divided by the product of 3 and 2"

A (5 ÷ 3) × 2(5 ÷ 3) × 2
B 3 × (2 ÷ 5)3 × (2 ÷ 5)
C (3 × 2) ÷ 5(3 × 2) ÷ 5
D 5 ÷ (3 × 2)

Answers

D]  5 ÷ (3 × 2) is the expression for the operation "5 divided by the product of 3 and 2".

The operation "5 divided by the product of 3 and 2" means that number 5 divided by the product of 3 and 2.

The mathematical representation of this operation is 5 ÷ (3 × 2).

The answer to this operation = 5 / 6.

Hence, 5 ÷ (3 × 2) is the expression for the operation "5 divided by the product of 3 and 2".

To understand more about multiplication and division refer -

https://brainly.com/question/28768606

#SPJ1

Differentiate a trig function that is greater than a power of 1, and involve either quotient, chain, or product rule.Differentiate a sine and cosine function that involves product and chain rule. Find the equation of the tangent line at x = a special triangle point (i.e. /4, /6, /3).Differentiate a function that involves both trig and exponential functions.[hint: add your own twist to this question for level 3/4]Differentiate an exponential function. [hint: add your own twist to this question for level 3/4]Differentiate a function where you have “y” and “x” on both sides of the equation and they cannot be simplified by collecting like terms or isolating y (i.e. y on one side and y^2 on the other). [hint: add your own twist to this question for level 3/4]

Answers

Solution:

Given a trigonometric function that is greater than power of 1 as shown below:

[tex]y=sin^2x\text{ ---- equation 1}[/tex]

To differentiate the function, we use the chain rule.

According to the chain rule,

[tex]\frac{dy}{dx}=\frac{dy}{du}\times\frac{du}{dx}[/tex]

From equation 1, let

[tex]u=sin\text{ x --- equation 2}[/tex]

This implies that

[tex]\begin{gathered} y=u^2 \\ \Rightarrow\frac{dy}{du}=2u \end{gathered}[/tex]

From equation 2,

[tex]\begin{gathered} \begin{equation*} u=sin\text{ x} \end{equation*} \\ \Rightarrow\frac{du}{dx}=cos\text{ x} \end{gathered}[/tex]

[tex]\begin{gathered} Recall\text{ from the chain rule:} \\ \frac{dy}{dx}=\frac{dy}{du}\times\frac{du}{dx} \\ \Rightarrow2u\times\text{cos x} \\ \frac{dy}{dx}=2ucos\text{ x} \\ but\text{ } \\ u=sin\text{ x} \\ \therefore\frac{dy}{dx}=2(sin\text{ }x)(cos\text{ }x) \end{gathered}[/tex]

Which point shows the number with the greatest absolute value? А B + + D TH 30 40 50 -50 -40 -30 - 20 -10 0 10 20 O Point A O Point B O Point C O Point D ہ

Answers

We are shown points A, B, C, and D on a number line.

We are asked to find out which point shows the number with the greatest absolute value?

Recall that the absolute value of a number is always positive.

The negative values of points A and B will become positive.

As you can see from the number line, point A is closer to -40 and the point D is closer to 30

The absolute value of point A will be closer to |-40| = 40

Since 40 is greater than 30, point A shows the number with the greatest absolute value.

Therefore, the correct answer is Point A.

Prison Sentences The average prison sentence for a person convicted of second-degree murder is 15 years. If the sentences are normally distributed with astandard deviation of 2.1 years, find the following probabilities.P (x> 18) =

Answers

Givens.

• The mean is 15 years.

,

• The standard deviation is 2.1 years.

,

• x = 18.

Using a graphic calculator, the probability P(X > 18) is 0.0766.

Therefore, the answer is 0.07

Moshde runs a hairstyling business from her house. She charges $42 for a haircut and style. Her monthly expenses are $1070. She wants to be able to put at least $1,249 per month into her savings account order to open her own salon. How many "cut & styles" must she do to save at least $1,249 per month?

Answers

ANSWER:

56 cut & styles

STEP-BY-STEP EXPLANATION:

They tell us that each haircut and style charges $42 and that the monthly expenses are $1070, he wants to save a total of $1249, with this information we can establish the following equation:

[tex]42x-1070=1249[/tex]

Where x would be the amount of cut & styles, we solve for x:

[tex]\begin{gathered} 42x=1249+1070 \\ x=\frac{2319}{42} \\ x=55.2 \\ x\cong56 \end{gathered}[/tex]

If you make a total of 55 cut & styles, the amount does not reach a total of $1249 per month, therefore, at least 56 cut & styles are needed, to achieve the monthly goal.

Hi, what is the LCM of the numbers 3 and 15

Answers

Answer:

15

Step-by-step Explanation:

LCM is the least common multiple, that is, is the least number that is a multiple of both numbers.

To find it, first, let's write the multiples of 3:

Multiples of 3: 3, 6, 9, 12, 15, 18, 21, 24, 27, 30, ...

Now, let's write the multiples of 15:

Multiples of 15: 15, 30, 45, ...

If you compare the multiples of 3 and 5, we can see that they have some common multiples, as 15 and 30.

From this common multiples, 15 is the smallest number. So, 15 is the LCM of 3 and 15.

Answer: 15.

h(x) =x² +9 if h(x)=9 , x =

Answers

The given expression as; h(x) =x² +9

for h(x) = 9

Substitute the value of h(x) = 9 in the given expression;

h(x) =x² +9

9 =x² +9

x² = 9 - 9

x² = 0

x = 0

Answer : x = 0

(4.7 x 10-3) x 351Simplify the expressionusing scientific notation and express your answer(2.5 x 10') < (3.3 X 100)in scientific notation. Round your answer to the nearest thousandth.AnswerKeypadKeyboard Shortcutsx10

Answers

Given:

[tex]\frac{(4.7\times10^{-3})\times351}{(2.5\times10^5)\times(3.3\times10^6)}[/tex]

Remove the brackets and multiply common terms

[tex]\begin{gathered} \frac{(4.7\times10^{-3})\times351}{(2.5\times10^5)\times(3.3\times10^6)} \\ =\frac{4.7\times10^{-3}\times351}{2.5\times10^5\times3.3\times10^6} \\ =\frac{4.7\times351\times10^{-3}}{2.5\times3.3\times10^5\times10^6} \\ =\frac{1649.7\times10^{-3}}{8.25\times10^{11}} \end{gathered}[/tex]

Simplify further to get

[tex]\begin{gathered} \frac{1649.7\times10^{-3}}{8.25\times10^{11}} \\ =\frac{16497\times10^{-1}_{}\times10^{-3}}{825\times10^{-2}\times10^{11}} \\ =\frac{16497\times10^{-4}}{825\times10^9} \end{gathered}[/tex]

This further gives

[tex]\begin{gathered} \frac{16497\times10^{-4}}{825\times10^9} \\ =\frac{16497}{825}\times\frac{10^{-4}}{10^9} \\ =19.996\times10^{-4-9} \\ =19.996\times10^{-14} \end{gathered}[/tex]

Therefore, the answer is

[tex]19.996\times10^{-14}[/tex]

An online company is advertising a mixer on sale for 35% off the original price of $224.99 what is the sale price for the mixer? Round your answer to the nearest cent, if necessary.

Answers

Given:

The original price of mixer is $224.99.

The discount on the mixer is 35%.

Explanation:

Determine the discount amount on the mixer.

[tex]\begin{gathered} d=\frac{35}{100}\cdot224.99 \\ =78.7465 \end{gathered}[/tex]

Determine the sale price of the mixer.

[tex]\begin{gathered} 224.99-78.7465=146.2435 \\ \approx146.24 \end{gathered}[/tex]

So sale price of the mixer is $146.24.

How many pounds of candy that sells for $0.85 per Ib must be mixed with candy that sells for $1.22 per lb to obtain 9 lb of a mixture that should sell for $0.92 per lb? 50.85-per-lb candy: 73 lb (Type an integer or decimal rounded to two decimal places as needed.) $1.22-per-b candy

Answers

This system gives two equations

[tex]\frac{0.85x+1.22y}{9}=0.92[/tex][tex]x+y=9[/tex]

where x is the number pounds of $0.85/lb candy and y is the number of pounds of $1.22/lb candy.

The solution to the system is

[tex]x=7.297[/tex][tex]y=1.70[/tex]

Hence, 7.297 lb of $0.85 candy is required in order that if we mix them with 1.70 lb of $1.22 candy, we will get a 9 lb solution of 0.92 /lb candy.

Vanessa collected Barbie dolls. She began with 2 dolls and added the same amount of dolls to her collection each year. In the 24th year, Vanessa had 98 dolls. Which function, d(n), can be used to determine the number of dolls Vanessa had in any year?

Answers

The correct answer is d(n) = 4n +2

in the diagram, ab to ec are perpendicular. if m

Answers

[tex]\begin{gathered} \text{sorry,} \\ 9x+13x+2=90 \\ 22x+2=90 \\ 22x=90-2 \\ 22x=88 \\ x=\frac{88}{22} \\ x=4 \end{gathered}[/tex][tex]\text{angle MEB=9x, since x=4, hnce, angle MEB=36}[/tex]

determine whether the binomial expression is a factor to the following polynomial.[tex]p(x) = {x}^{3} - 9x + 1 \: \: \: \: \: \: \: \: \: (x - 3)[/tex]the binomial expression is (x-3) ^^answer choicesA. yesB. no

Answers

We can find if (x-3) is a factor by dividing P(x) by (x-3).

A simpler way is replacing x with 3 and if the value of P(x) is 0, then (x-3) is a factor of P(x). This is because x=3 is a root of P(x) and therefore it can be factorized with the term (x-3).

Then, we calculate P(3):

[tex]P(3)=3^2-9\cdot3+1=9-27+1=-17[/tex]

As x=3 is not a root of P(x), then (x-3) is not a factor of P(x).

Answer: No.

Amelia bought spider rings for Halloween goodie bags. She bought 13 packs of red rings, 16 packs of yellow rings, and 14 packs of green rings. If each pack had 12 rings, how many rings did Amelia buy?

Answers

We know that

• She bought 13 packs of red rings.

,

• She bought 16 packs of yellow rings.

,

• She bought 14 packs of green rings.

,

• Each pack has 12 rings.

This problem is about multiplication, notice that each pack includes 12 rings, that means we need to multiply each pack by 12, in order to find the total number of rings of each color.

[tex]R=13\cdot12=156[/tex]

There are 156 red rings.

[tex]Y=16\cdot12=192[/tex]

There are 192 yellow rings.

[tex]G=14\cdot12=168[/tex]

There are 168 green rings.

Now, we sum all these numbers to find the total

[tex]T=168+192+156=516[/tex]Therefore, there are 516 rings in total.

Translate to a system Reiko needs to mail her Christmas cards and packages and wants to keep her mailing costs to no more than $500. The number of cards is at least 4 more than twice the number of packages. The cost of mailing a card (with pictures enclosed) is $3 and for a package the cost is $7.

Answers

Given:

Let x be the number of the cards and y be the number of the package.

Given that the number of cards is at least 4 more than twice the number of packages.

[tex]x=2y+4[/tex]

Given that mailing costs no more than $500 and the cost of mailing a card is $3 and for a package, the cost is $7.

[tex]3x+7y=500[/tex]

Substitute x=2y+4 in this equation, we get

[tex]3(2y+4)+7y=500[/tex]

[tex]6y+8+7y=500[/tex]

[tex]13y=500-8[/tex]

[tex]y=\frac{492}{13}[/tex][tex]y=37.8[/tex]

Let y=37 and substitute in x=2y+4, we get

[tex]x=2\times37+4[/tex][tex]x=78[/tex]

Hence the number of cards = 78 and the number of packages =37.

The total cost for this is $493 not more than $500.

A 51-inch TV suggests that the main diagonal of the TV is 51 inches. Determine the dimensions of the screen of a 51 -inch TV with a 16:9 aspect ratio.Please see attached photo

Answers

The aspect ratio 16:9 indicates the next relation between x and y:

[tex]\frac{y}{x}=\frac{16}{9}[/tex]

Applying the Pythagorean theorem to the right triangle formed:

[tex]51^2=x^2+y^2[/tex]

Isolating y from the first equation:

[tex]y=\frac{16}{9}x[/tex]

Substituting in the second equation:

[tex]\begin{gathered} 51^2=x^2+(\frac{16}{9}x)^2 \\ 2601=x^2+(\frac{16}{9})^2x^2 \\ 2601=x^2+\frac{16^2}{9^2}^{}x^2 \\ 2601=x^2+\frac{256}{81}^{}x^2 \\ 2601=\frac{337}{81}^{}x^2 \\ 2601\cdot\frac{81}{337}=x^2 \\ 625.166172=x^2 \\ \sqrt[]{625.17}\approx x \\ 25\approx x \end{gathered}[/tex]

Replacing in the equation of y:

[tex]\begin{gathered} y=\frac{16}{9}\cdot25 \\ y\approx44.44 \end{gathered}[/tex]

The approximate dimensions are:

length = 25 in

height = 44.44 in

Annie's backyard deck cost $61.75 per square meter to build. The deck is 7 meters wide and14 meters long. How much did it cost to build the deck?

Answers

ANSWER

the cost to build the deck is $6051.5

EXPLANATION

Given that;

The length of the deck is 14 m

The width of the deck is 7m

1 m^2 is equivalent to $61.75

Follow the steps below to find the cost to build the deck

Step 1; Find the area of the deck

[tex]\begin{gathered} \text{ Recall, that the deck is a rectangular shape} \\ \text{ Area of a rectangle = length }\times\text{ width} \\ \text{ Area of a reactangle = 14 }\times\text{ 7} \\ \text{ Area of a rectangle = 98m}^2 \end{gathered}[/tex]

Step 2; Find the total cost of the deck

Let x represents the total cost to build the deck

[tex]\begin{gathered} \text{ 1m}^2\text{ }\rightarrow\text{ \$61.75} \\ \text{ 98m}^2\text{ }\rightarrow\text{ \$x} \\ \text{ cross multiply} \\ \text{ 1m}^2\text{ }\times\text{ \$x = \$61.75 }\times\text{ 98m}^2 \\ \text{ Isolate \$x }\frac{}{} \\ \text{ \$x = }\frac{\text{ \$61.75}\times98\cancel{m^2}}{1\cancel{m^2}} \\ \text{ \$x = \$61.75 }\times\text{ 98} \\ \text{ \$x = \$6051.5} \end{gathered}[/tex]

Therefore, the cost to build the deck is $6051.5

Can someone please help me find the value of X?

Answers

Remember that

the sum of the interior angles in any polygon is equal to

S=180(n-2)

where

n is the number of sides of polygon

In this problem

we have

n=6 (hexagon)

so

substitute

S=180(6-2)

S=720 degrees

step 2

Adds the interior angles

720=120+(5x-6)+(4x+14)+(7x)+(8x-8)+(6x)

solve for x

combine like terms

720=30x+120

30x=720-120

30x=600

x=20

3x and 8x are like terms.true or false

Answers

Like terms are those terms whose variable and its corresponding exponent are the same. Here we have 3x and 8x. Both terms have the number:

[tex]x^1[/tex]

Which means that they have the same variable and the same exponents. Then they are like terms and the answer is True.

find the volume round to the nearest tenth use 3.14 for pi 5km

Answers

Step 1

List all parameters

[tex]\begin{gathered} \pi\text{ = 3.14} \\ r\text{ = 5km} \\ \end{gathered}[/tex]

Step 2

Write the volume of a sphere

[tex]undefined[/tex]

3. A toy box is 24 cm long, 15 cm wide and 11 cm high. What is the volume of the toy box? What is the correct number sentence for this problem? A.V=24×15×11B.V=24×15C.V=24×11D.V=15×11

Answers

ANSWER

[tex]\begin{gathered} V=24*15*11 \\ V=3960\text{ }cm^3 \end{gathered}[/tex]

EXPLANATION

The box is a rectangular prism. The volume of a rectangular prism is given by:

[tex]V=L*W*H[/tex]

where L = length

W = width

H = height

Therefore, the volume of the box can be written in the number sentence:

[tex]V=24*15*11[/tex]

and the volume of the box is:

[tex]V=3960\text{ }cm^3[/tex]

That is the answer.

Find the height of the cliff. If necessary, round to the nearest hundredth yard.

Answers

We are given a diagram showing a slope, and a vertical height. We now have what represents a right angled triangle. The distance from the base of the cliff to the end of the slope is given as 24 yards. The slope itself is 37 yards. We shall now determine the height of the cliff (from ground to top) as indicated.

Note that we shall use the Pythagoras' theorem which is;

[tex]c^2=a^2+b^2[/tex]

Where we have

[tex]\begin{gathered} c=\text{hypotenuse (longest side)} \\ a,b=\text{other sides} \end{gathered}[/tex]

We can now substitute the given values/side lengths and we'll have;

[tex]37^2=24^2+b^2[/tex][tex]1369=576+b^2[/tex]

Subtract 576 from both sides;

[tex]793=b^2[/tex]

Take the square root of both sides;

[tex]\begin{gathered} \sqrt[]{793}=\sqrt[]{b^2} \\ 28.160255\ldots=b \end{gathered}[/tex]

Rounded to the nearest hundredth, the answer now becomes;

ANSWER:

[tex]b=28.16yd[/tex]

The last option is the correct answer

1/9=_/54What is the answer?

Answers

[tex]\begin{gathered} \frac{1}{9}=\frac{x}{54} \\ \frac{1}{9}\cdot54=\frac{x}{54}\cdot54 \\ \frac{54}{9}=x \\ \\ 6=x \\ \\ \\ \text{thus} \\ \frac{1}{9}=\frac{6}{54} \end{gathered}[/tex]

the line contains the point (-3,5) and is perpendicular to the line y=3x-4

Answers

two lines are perpendicular when the multiplication of their slopes is equal to -1. The slope of y = 3x - 4 is 3. Then the slope of a perpendicular line ​is:

[tex]\begin{gathered} m\cdot3=-1 \\ m=-\frac{1}{3} \end{gathered}[/tex]

Slope-intercept form:

y = mx + b

where m is the slope and b is the y-intercept. Replacing with point (-3, 5) and m = -1/3, we get:

5 = -1/3(-3) + b

5 = 1+ b

5 - 1 = b

4 = b

Then, the equation is:

y = -1/3x + 4

the relationship between the minutes a candle is burned and the size of the candle in millimeters is shown on the graph.

Answers

The function is a decreasin line so the more time goes the side will decrease so the correct answer is:

The candle started at 9mm and shrinks 5mm every 4 minutes

Callie's grandmother pledged R150, 00 for every mile Callie walked in her walk-a-thon. Callie walked 14.5 km. How much does her grandmother owe? ( assume 8 km = 5miles)

Answers

Given:-

Callie walked 14.5 km. And also given 8 km=5 miles.

At first we convert 14.5 km to miles. we get,

[tex]\begin{gathered} 14.5\times\frac{5}{8}=\frac{72.5}{8} \\ \text{ =9.0625} \end{gathered}[/tex]

So 14.5 km is 9.0625 miles.

Callie's grandmother pledged Rs. 150 for every mile. so for 9.0625 miles it is,

[tex]9.0625\times150=1359.375[/tex]

So her grandmother owe Rs. 1359.375

1. What is the other endpoint of the segment with midpoint -3 and endpoint -7? A-11 01 D 4 B -5 2. The endngints of ST are S(2,-2) and T14, 2). What are the coordinates of the

Answers

We have a segment with points S and T.

We know the coordinates of S=(2,-2) and the midpoint M=(-

Type the correct answer in the box. Use numericals instead of words. 5 less than a number is equivalent to 1 more than three times the number. The number is _____.

Answers

Answer:

2

Explanation:

Let the number be x

5 less than a number is expressed as x - 5

1 more than three times the number is expressed as 3x + 1

Equate both expression and find the number

x - 5 = 3x+1

x - 3x = 1 - 5

-2x = -4

x = -4/-2

x = 2

Hence the number is 2

the sum of 6 times a number and 8 equals 7? translate into equation

Answers

Let:

x = Unknown number

the sum of 6 times a number:

[tex]6x+[/tex]

and 8 equals 7, so:

[tex]\begin{gathered} 6x+8=7 \\ \text{solve for x:} \\ \text{subtract 8 from both sides:} \\ 6x=7-8 \\ 6x=-1 \\ \text{divide both sides by 6:} \\ x=-\frac{1}{6} \end{gathered}[/tex]

Other Questions
URGENT!! ILL GIVEBRAINLIEST!!!! AND 100 POINTS!!!!!! GEOMETRY: Express the volume of each cube below as a monomialNeeded fast! Is (2,7) a solution to the system of equations: y= x + 5 y= 4x + 1 Part 1 (circle 1) Yes No Part 2 Shows a proof of your answer What is the momentum of a 0.144kg baseball thrown at a speed of 46m/s? Give your answer in kgm/s. I have been stuck on this for a while, your help would be most appreciated! How do I know if 6.209 is greater or lesser than 6.29 Three cards are drawn from an ordinary deck and not replaced find the probability of the following P(getting 3 queens)P(getting an ace and king and queen)P(getting club and spade and heart)P(getting 3 diamonds)Answer the following problems using multiplication rule make sure to reduce your fraction A regular hexagon has perimeter 72m. Find the area DNA is replicated before A. crossing -over B. cell division c. cell death D. transformation. please help!! 100pts and brainliest! Rb+ Fe2+ H- Al3+ Cl- O2-Using the ions above, write the formulas and give the names for 5 possible ionic compounds involving these ions. If f(x)=x-20 and g(x) = 4+3x, then f(g(-3)) = The solutions to a quadratic equation are -2 and 6. What is the equation of its axisof symmetry? The sum of two numbers is30. The sum of4 times the larger and6 times the smaller is128. Find the numbers. What type of number is 3 - 77iChoose all answers that apply:A. RealB. ImaginaryC. Complex Question: What is the solution to this system of linear equations? 8x + 2y = 2 and x + 3y 2 = 14 I NEED The claim, evidence and reasoning. Please I need help Blair purchases the T-shirts from Company B. She needs to add a 75markup to her total cost to make a profit when she sells the shirts at thcarnival. How should Blair determine the selling price of each T-shirt athe carnival?Drag a number into each box to make the statements true.She canThe total cost Blair paid for one T-shirt is $calculate the markup by multiplying her total cost byBlair will sell each T-shirt at the carnival for $ help my brother the other one I tried was a scam find the slope of the line A dozen muffins is 12 muffins. A recipe calls for 3/4 cup of almonds to make half a dozen muffins. Use a complex fraction to find how many cups of almonds are needed to make a dozen muffins. why short circuit are dangrous